jreeve12
Thanks Received: 0
Forum Guests
 
Posts: 4
Joined: May 08th, 2013
 
 
 

Q22 - Despite improvements in the treatment

by jreeve12 Thu May 23, 2013 11:49 pm

I eliminated answer choices (C), (D), and (E) fairly easily.

However, when down to (B) and (A), I failed to distinguish between the two answers and guessed (incorrectly) for (B).

(B) states: "Records of asthma dates are as accurate for the past twenty years as for the past ten years."

I don't see how this strengthens the argument, as the very first sentence confines the scope of the argument to "the death rate from this disease" which has "doubled during the past decade from its previous rate."

Why should we care how accurate records for the past twenty years were? We're only interested in what caused the increase duringthe decade under consideration. Moreover, the stimulus seems to directly contradict (B): "Two possible explanations for this increase have been offered. First, the recording of deaths due to asthma has become more widespread and accurate in the past decade than it had been previously."

What am I missing here?
 
zagreus77
Thanks Received: 10
Jackie Chiles
Jackie Chiles
 
Posts: 34
Joined: May 01st, 2012
 
 
 

Re: Q22 - Despite improvements in the treatment for asthma, the

by zagreus77 Fri May 24, 2013 8:18 pm

jreeve12 Wrote:I eliminated answer choices (C), (D), and (E) fairly easily.

However, when down to (B) and (A), I failed to distinguish between the two answers and guessed (incorrectly) for (B).

(B) states: "Records of asthma dates are as accurate for the past twenty years as for the past ten years."

I don't see how this strengthens the argument, as the very first sentence confines the scope of the argument to "the death rate from this disease" which has "doubled during the past decade from its previous rate."

Why should we care how accurate records for the past twenty years were? We're only interested in what caused the increase duringthe decade under consideration. Moreover, the stimulus seems to directly contradict (B): "Two possible explanations for this increase have been offered. First, the recording of deaths due to asthma has become more widespread and accurate in the past decade than it had been previously."

What am I missing here?

It knocks one of the competitor explanations that records over the past decade..... This is a tough stimuli, scary to me, really, but once I read the choices, it wasn't as bad. I saw b and realized it couldn't coexist with one of the explanations.
 
agersh144
Thanks Received: 6
Elle Woods
Elle Woods
 
Posts: 84
Joined: December 20th, 2012
 
 
 

Re: Q22 - Despite improvements in the treatment

by agersh144 Thu Jul 18, 2013 11:22 pm

I still don't understand the stimulus says that records of deaths due to asthma have becomes more widespread and accurate in the past decade than it had been previous. B says NO actually records of asthma deaths are AS accurate for the past twenty years as for the past tens year. This directly contradicts the stimulus. This is not supporting the stimulus I'm just completely flabbergasted right now.
User avatar
 
ManhattanPrepLSAT1
Thanks Received: 1909
Atticus Finch
Atticus Finch
 
Posts: 2851
Joined: October 07th, 2009
 
This post thanked 1 time.
 
 

Re: Q22 - Despite improvements in the treatment

by ManhattanPrepLSAT1 Mon Jul 22, 2013 5:22 pm

Great discussion so far!

Lets start with the argument in the stimulus. It cites a phenomenon (increased death rate from asthma), presents two possible explanations (better reporting and urban pollution) of the phenomenon, criticizes the two explanations presented, and then offers what the author considers to be the real explanation (bronchial inhalers) of the phenomenon.

The question asks us to find the one answer choice that does not offer support to the argument--that means the four that support the argument should be eliminated.

Answer choice (A) is irrelevant to the argument because the death rate from asthma has increased in areas with long-standing track records of accurate reporting and little to no pollution. Increasing the size of the urban population does not affect these two considerations we must accept as fact.

Incorrect Answers
(B) supports the claim that medical records are accurate and so suggests that something else must explain the increased death rate. This may difficult to see as relevant, but the critical language is that the rate as double this decade from its previous rate--meaning the death rate in the last decade was up from the death rate in previous decades. If in previous decades the accuracy of medical records was not as accurate, it could be that the more accurate medical records of this past decade could explain why the death rate doubled.
(C) and (D) provide means by which bronchial inhalers could create a problem for asthma sufferers.
(E) shows how bronchial inhalers could be the culprit for the change in the death rate from asthma--though it doesn't show why, it does point the finger at bronchial inhalers.
User avatar
 
Mab6q
Thanks Received: 31
Atticus Finch
Atticus Finch
 
Posts: 290
Joined: June 30th, 2013
 
This post thanked 1 time.
 
 

Re: Q22 - Despite improvements in the treatment

by Mab6q Tue Oct 14, 2014 6:56 pm

Actually, I believe A weakens the argument. It shows us that the urban population has doubled during the same time as the death rate from asthma has doubled, presenting an alternative cause for the increase.
"Just keep swimming"